Multi peaks fit (Voigt, Lorentzian or Gaussian)












4














I have started quite recently to use Mathematica and I don´t have experience in coding.



What I need to do is to fit a peak which probably is a combination of two Voigt or Lorentzian. I have tried with code already present in the forum but without big success. Could you help me in writing this? Maybe with comments in the code so I can understand better what we are doing.



Data



data = {{19.4, 16672.}, {19.41, 16642.}, {19.42, 16778.}, {19.43, 
16857.}, {19.44, 16833.}, {19.45, 17086.}, {19.46, 17129.}, {19.47,
17405.}, {19.48, 17483.}, {19.49, 17308.}, {19.5, 17884.}, {19.51,
17950.}, {19.52, 18202.}, {19.53, 18473.}, {19.54, 19021.}, {19.55,
19279.}, {19.56, 20040.}, {19.57, 20399.}, {19.58, 21412.}, {19.59,
22354.}, {19.6, 23334.}, {19.61, 24399.}, {19.62, 25724.}, {19.63,
27133.}, {19.64, 28825.}, {19.65, 30078.}, {19.66, 32224.}, {19.67,
33907.}, {19.68, 36299.}, {19.69, 39152.}, {19.7, 41980.}, {19.71,
45181.}, {19.72, 49547.}, {19.73, 55438.}, {19.74, 62094.}, {19.75,
69884.}, {19.76, 80306.}, {19.77, 92448.}, {19.78, 107115.}, {19.79,
126574.}, {19.8, 148842.}, {19.81, 175298.}, {19.82,
205953.}, {19.83, 240900.}, {19.84, 278834.}, {19.85,
322364.}, {19.86, 365952.}, {19.87, 411105.}, {19.88,
457658.}, {19.89, 500221.}, {19.9, 544824.}, {19.91,
583862.}, {19.92, 619383.}, {19.93, 650362.}, {19.94,
672886.}, {19.95, 690179.}, {19.96, 695603.}, {19.97,
692265.}, {19.98, 677707.}, {19.99, 657226.}, {20.,
630722.}, {20.01, 599184.}, {20.02, 558854.}, {20.03,
514989.}, {20.04, 469037.}, {20.05, 421656.}, {20.06,
370503.}, {20.07, 324609.}, {20.08, 278435.}, {20.09,
233750.}, {20.1, 195167.}, {20.11, 160965.}, {20.12,
131452.}, {20.13, 108026.}, {20.14, 88341.}, {20.15,
71993.}, {20.16, 59909.}, {20.17, 51054.}, {20.18, 44365.}, {20.19,
39526.}, {20.2, 36292.}, {20.21, 34308.}, {20.22, 32666.}, {20.23,
31599.}, {20.24, 30743.}, {20.25, 29621.}, {20.26, 29034.}, {20.27,
28213.}, {20.28, 27597.}, {20.29, 27485.}, {20.3, 26921.}, {20.31,
26588.}, {20.32, 26337.}, {20.33, 25705.}, {20.34, 26199.}, {20.35,
25321.}, {20.36, 25017.}, {20.37, 25011.}, {20.38, 24566.}, {20.39,
24232.}, {20.4, 24005.}}


My starting point is



data1 = Rest@Transpose[Rescale /@ (Transpose@data)];
peakfunc[A_, [Mu]_, [Sigma]_, x_] = A^2 E^(-((x - [Mu])^2/(2 [Sigma]^2)));

Clear[model, modelvalue]
model[data_, n_] :=
Module[{dataconfig, modelfunc, objfunc, fitvar, fitres},
dataconfig = {A[#], [Mu][#], [Sigma][#]} & /@ Range[n];
modelfunc = (peakfunc[##, fitvar] & @@@ dataconfig // Total);
objfunc =
Total[((Sqrt[data[[All, 2]]])/
data[[All,
1]]) (data[[All, 2]] - (modelfunc /. fitvar -> # &) /@
data[[All, 1]])^2];
FindMinimum[objfunc, Join[{}, Flatten@dataconfig]]]
modelvalue[data_, n_] /; NumericQ[n] :=
If[n >= 1, model[data, n][[1]], 0]
fitres = ReleaseHold[
Hold[{Round[n], model[data1, Round[n]]}] /.
FindMinimum[modelvalue[data1, Round[n]], {n, 5},
Method -> "PrincipalAxis"][[2]]] // Quiet

With[{n = 2},
resfunc =
peakfunc[A[#], [Mu][#], [Sigma][#], x] & /@ Range[n] /.
model[data1, n][[2]]]

Show@{Plot[Evaluate[resfunc], {x, 0, 1},
PlotStyle -> ({Directive[Dashed, Thick,
ColorData["Rainbow"][#]]} & /@
Rescale[Range[Length[resfunc]]]), PlotRange -> All,
Frame -> True, Axes -> False],
Plot[Evaluate[Total@resfunc], {x, 0, 1},
PlotStyle -> Directive[Thick, Red], PlotRange -> All,
Frame -> True, Axes -> False],
Graphics[{PointSize[.003], Black, Point@data1}]}


This is already not very clear to me, as you can see the fit is not very good on the right side (I expect two profiles very close as the yellow and green curve in the second picture). I would like also to know how to evaluate if the fit is good enough or not.



enter image description here



enter image description here



Many thanks!










share|improve this question









New contributor




Clarine is a new contributor to this site. Take care in asking for clarification, commenting, and answering.
Check out our Code of Conduct.
















  • 2




    Welcome on Mathematic.StackExchange. Maybe NonlinearModelFit might be helpful to you? At least, make sure to have a look at its documentation
    – Henrik Schumacher
    yesterday






  • 3




    As a reminder, make sure you have good initial guesses for the parameters of your fitting function.
    – J. M. is computer-less
    yesterday






  • 1




    Are the integer values counts or measurements that just recorded as integers? I ask because it is important to know if what you have is a random sample from a maybe not-so-simple probability distribution or you're performing a regression that just might happen to have the shape of a mixture of probability distributions? In other words, are there 16,928,270 observations given as a histogram? (I'm also curious as to how you might have a "feeling" about the shape of the distributions - but that's not a Mathematica issue.)
    – JimB
    yesterday












  • This question seems to be a duplicate of 98226.
    – Anton Antonov
    yesterday












  • If you do have a histogram of counts, I would have to assume that you've got a truncated sample (truncated on the left at 19.395 and truncated on the right at 20.405). If so, please add that information to the body of your question.
    – JimB
    yesterday
















4














I have started quite recently to use Mathematica and I don´t have experience in coding.



What I need to do is to fit a peak which probably is a combination of two Voigt or Lorentzian. I have tried with code already present in the forum but without big success. Could you help me in writing this? Maybe with comments in the code so I can understand better what we are doing.



Data



data = {{19.4, 16672.}, {19.41, 16642.}, {19.42, 16778.}, {19.43, 
16857.}, {19.44, 16833.}, {19.45, 17086.}, {19.46, 17129.}, {19.47,
17405.}, {19.48, 17483.}, {19.49, 17308.}, {19.5, 17884.}, {19.51,
17950.}, {19.52, 18202.}, {19.53, 18473.}, {19.54, 19021.}, {19.55,
19279.}, {19.56, 20040.}, {19.57, 20399.}, {19.58, 21412.}, {19.59,
22354.}, {19.6, 23334.}, {19.61, 24399.}, {19.62, 25724.}, {19.63,
27133.}, {19.64, 28825.}, {19.65, 30078.}, {19.66, 32224.}, {19.67,
33907.}, {19.68, 36299.}, {19.69, 39152.}, {19.7, 41980.}, {19.71,
45181.}, {19.72, 49547.}, {19.73, 55438.}, {19.74, 62094.}, {19.75,
69884.}, {19.76, 80306.}, {19.77, 92448.}, {19.78, 107115.}, {19.79,
126574.}, {19.8, 148842.}, {19.81, 175298.}, {19.82,
205953.}, {19.83, 240900.}, {19.84, 278834.}, {19.85,
322364.}, {19.86, 365952.}, {19.87, 411105.}, {19.88,
457658.}, {19.89, 500221.}, {19.9, 544824.}, {19.91,
583862.}, {19.92, 619383.}, {19.93, 650362.}, {19.94,
672886.}, {19.95, 690179.}, {19.96, 695603.}, {19.97,
692265.}, {19.98, 677707.}, {19.99, 657226.}, {20.,
630722.}, {20.01, 599184.}, {20.02, 558854.}, {20.03,
514989.}, {20.04, 469037.}, {20.05, 421656.}, {20.06,
370503.}, {20.07, 324609.}, {20.08, 278435.}, {20.09,
233750.}, {20.1, 195167.}, {20.11, 160965.}, {20.12,
131452.}, {20.13, 108026.}, {20.14, 88341.}, {20.15,
71993.}, {20.16, 59909.}, {20.17, 51054.}, {20.18, 44365.}, {20.19,
39526.}, {20.2, 36292.}, {20.21, 34308.}, {20.22, 32666.}, {20.23,
31599.}, {20.24, 30743.}, {20.25, 29621.}, {20.26, 29034.}, {20.27,
28213.}, {20.28, 27597.}, {20.29, 27485.}, {20.3, 26921.}, {20.31,
26588.}, {20.32, 26337.}, {20.33, 25705.}, {20.34, 26199.}, {20.35,
25321.}, {20.36, 25017.}, {20.37, 25011.}, {20.38, 24566.}, {20.39,
24232.}, {20.4, 24005.}}


My starting point is



data1 = Rest@Transpose[Rescale /@ (Transpose@data)];
peakfunc[A_, [Mu]_, [Sigma]_, x_] = A^2 E^(-((x - [Mu])^2/(2 [Sigma]^2)));

Clear[model, modelvalue]
model[data_, n_] :=
Module[{dataconfig, modelfunc, objfunc, fitvar, fitres},
dataconfig = {A[#], [Mu][#], [Sigma][#]} & /@ Range[n];
modelfunc = (peakfunc[##, fitvar] & @@@ dataconfig // Total);
objfunc =
Total[((Sqrt[data[[All, 2]]])/
data[[All,
1]]) (data[[All, 2]] - (modelfunc /. fitvar -> # &) /@
data[[All, 1]])^2];
FindMinimum[objfunc, Join[{}, Flatten@dataconfig]]]
modelvalue[data_, n_] /; NumericQ[n] :=
If[n >= 1, model[data, n][[1]], 0]
fitres = ReleaseHold[
Hold[{Round[n], model[data1, Round[n]]}] /.
FindMinimum[modelvalue[data1, Round[n]], {n, 5},
Method -> "PrincipalAxis"][[2]]] // Quiet

With[{n = 2},
resfunc =
peakfunc[A[#], [Mu][#], [Sigma][#], x] & /@ Range[n] /.
model[data1, n][[2]]]

Show@{Plot[Evaluate[resfunc], {x, 0, 1},
PlotStyle -> ({Directive[Dashed, Thick,
ColorData["Rainbow"][#]]} & /@
Rescale[Range[Length[resfunc]]]), PlotRange -> All,
Frame -> True, Axes -> False],
Plot[Evaluate[Total@resfunc], {x, 0, 1},
PlotStyle -> Directive[Thick, Red], PlotRange -> All,
Frame -> True, Axes -> False],
Graphics[{PointSize[.003], Black, Point@data1}]}


This is already not very clear to me, as you can see the fit is not very good on the right side (I expect two profiles very close as the yellow and green curve in the second picture). I would like also to know how to evaluate if the fit is good enough or not.



enter image description here



enter image description here



Many thanks!










share|improve this question









New contributor




Clarine is a new contributor to this site. Take care in asking for clarification, commenting, and answering.
Check out our Code of Conduct.
















  • 2




    Welcome on Mathematic.StackExchange. Maybe NonlinearModelFit might be helpful to you? At least, make sure to have a look at its documentation
    – Henrik Schumacher
    yesterday






  • 3




    As a reminder, make sure you have good initial guesses for the parameters of your fitting function.
    – J. M. is computer-less
    yesterday






  • 1




    Are the integer values counts or measurements that just recorded as integers? I ask because it is important to know if what you have is a random sample from a maybe not-so-simple probability distribution or you're performing a regression that just might happen to have the shape of a mixture of probability distributions? In other words, are there 16,928,270 observations given as a histogram? (I'm also curious as to how you might have a "feeling" about the shape of the distributions - but that's not a Mathematica issue.)
    – JimB
    yesterday












  • This question seems to be a duplicate of 98226.
    – Anton Antonov
    yesterday












  • If you do have a histogram of counts, I would have to assume that you've got a truncated sample (truncated on the left at 19.395 and truncated on the right at 20.405). If so, please add that information to the body of your question.
    – JimB
    yesterday














4












4








4







I have started quite recently to use Mathematica and I don´t have experience in coding.



What I need to do is to fit a peak which probably is a combination of two Voigt or Lorentzian. I have tried with code already present in the forum but without big success. Could you help me in writing this? Maybe with comments in the code so I can understand better what we are doing.



Data



data = {{19.4, 16672.}, {19.41, 16642.}, {19.42, 16778.}, {19.43, 
16857.}, {19.44, 16833.}, {19.45, 17086.}, {19.46, 17129.}, {19.47,
17405.}, {19.48, 17483.}, {19.49, 17308.}, {19.5, 17884.}, {19.51,
17950.}, {19.52, 18202.}, {19.53, 18473.}, {19.54, 19021.}, {19.55,
19279.}, {19.56, 20040.}, {19.57, 20399.}, {19.58, 21412.}, {19.59,
22354.}, {19.6, 23334.}, {19.61, 24399.}, {19.62, 25724.}, {19.63,
27133.}, {19.64, 28825.}, {19.65, 30078.}, {19.66, 32224.}, {19.67,
33907.}, {19.68, 36299.}, {19.69, 39152.}, {19.7, 41980.}, {19.71,
45181.}, {19.72, 49547.}, {19.73, 55438.}, {19.74, 62094.}, {19.75,
69884.}, {19.76, 80306.}, {19.77, 92448.}, {19.78, 107115.}, {19.79,
126574.}, {19.8, 148842.}, {19.81, 175298.}, {19.82,
205953.}, {19.83, 240900.}, {19.84, 278834.}, {19.85,
322364.}, {19.86, 365952.}, {19.87, 411105.}, {19.88,
457658.}, {19.89, 500221.}, {19.9, 544824.}, {19.91,
583862.}, {19.92, 619383.}, {19.93, 650362.}, {19.94,
672886.}, {19.95, 690179.}, {19.96, 695603.}, {19.97,
692265.}, {19.98, 677707.}, {19.99, 657226.}, {20.,
630722.}, {20.01, 599184.}, {20.02, 558854.}, {20.03,
514989.}, {20.04, 469037.}, {20.05, 421656.}, {20.06,
370503.}, {20.07, 324609.}, {20.08, 278435.}, {20.09,
233750.}, {20.1, 195167.}, {20.11, 160965.}, {20.12,
131452.}, {20.13, 108026.}, {20.14, 88341.}, {20.15,
71993.}, {20.16, 59909.}, {20.17, 51054.}, {20.18, 44365.}, {20.19,
39526.}, {20.2, 36292.}, {20.21, 34308.}, {20.22, 32666.}, {20.23,
31599.}, {20.24, 30743.}, {20.25, 29621.}, {20.26, 29034.}, {20.27,
28213.}, {20.28, 27597.}, {20.29, 27485.}, {20.3, 26921.}, {20.31,
26588.}, {20.32, 26337.}, {20.33, 25705.}, {20.34, 26199.}, {20.35,
25321.}, {20.36, 25017.}, {20.37, 25011.}, {20.38, 24566.}, {20.39,
24232.}, {20.4, 24005.}}


My starting point is



data1 = Rest@Transpose[Rescale /@ (Transpose@data)];
peakfunc[A_, [Mu]_, [Sigma]_, x_] = A^2 E^(-((x - [Mu])^2/(2 [Sigma]^2)));

Clear[model, modelvalue]
model[data_, n_] :=
Module[{dataconfig, modelfunc, objfunc, fitvar, fitres},
dataconfig = {A[#], [Mu][#], [Sigma][#]} & /@ Range[n];
modelfunc = (peakfunc[##, fitvar] & @@@ dataconfig // Total);
objfunc =
Total[((Sqrt[data[[All, 2]]])/
data[[All,
1]]) (data[[All, 2]] - (modelfunc /. fitvar -> # &) /@
data[[All, 1]])^2];
FindMinimum[objfunc, Join[{}, Flatten@dataconfig]]]
modelvalue[data_, n_] /; NumericQ[n] :=
If[n >= 1, model[data, n][[1]], 0]
fitres = ReleaseHold[
Hold[{Round[n], model[data1, Round[n]]}] /.
FindMinimum[modelvalue[data1, Round[n]], {n, 5},
Method -> "PrincipalAxis"][[2]]] // Quiet

With[{n = 2},
resfunc =
peakfunc[A[#], [Mu][#], [Sigma][#], x] & /@ Range[n] /.
model[data1, n][[2]]]

Show@{Plot[Evaluate[resfunc], {x, 0, 1},
PlotStyle -> ({Directive[Dashed, Thick,
ColorData["Rainbow"][#]]} & /@
Rescale[Range[Length[resfunc]]]), PlotRange -> All,
Frame -> True, Axes -> False],
Plot[Evaluate[Total@resfunc], {x, 0, 1},
PlotStyle -> Directive[Thick, Red], PlotRange -> All,
Frame -> True, Axes -> False],
Graphics[{PointSize[.003], Black, Point@data1}]}


This is already not very clear to me, as you can see the fit is not very good on the right side (I expect two profiles very close as the yellow and green curve in the second picture). I would like also to know how to evaluate if the fit is good enough or not.



enter image description here



enter image description here



Many thanks!










share|improve this question









New contributor




Clarine is a new contributor to this site. Take care in asking for clarification, commenting, and answering.
Check out our Code of Conduct.











I have started quite recently to use Mathematica and I don´t have experience in coding.



What I need to do is to fit a peak which probably is a combination of two Voigt or Lorentzian. I have tried with code already present in the forum but without big success. Could you help me in writing this? Maybe with comments in the code so I can understand better what we are doing.



Data



data = {{19.4, 16672.}, {19.41, 16642.}, {19.42, 16778.}, {19.43, 
16857.}, {19.44, 16833.}, {19.45, 17086.}, {19.46, 17129.}, {19.47,
17405.}, {19.48, 17483.}, {19.49, 17308.}, {19.5, 17884.}, {19.51,
17950.}, {19.52, 18202.}, {19.53, 18473.}, {19.54, 19021.}, {19.55,
19279.}, {19.56, 20040.}, {19.57, 20399.}, {19.58, 21412.}, {19.59,
22354.}, {19.6, 23334.}, {19.61, 24399.}, {19.62, 25724.}, {19.63,
27133.}, {19.64, 28825.}, {19.65, 30078.}, {19.66, 32224.}, {19.67,
33907.}, {19.68, 36299.}, {19.69, 39152.}, {19.7, 41980.}, {19.71,
45181.}, {19.72, 49547.}, {19.73, 55438.}, {19.74, 62094.}, {19.75,
69884.}, {19.76, 80306.}, {19.77, 92448.}, {19.78, 107115.}, {19.79,
126574.}, {19.8, 148842.}, {19.81, 175298.}, {19.82,
205953.}, {19.83, 240900.}, {19.84, 278834.}, {19.85,
322364.}, {19.86, 365952.}, {19.87, 411105.}, {19.88,
457658.}, {19.89, 500221.}, {19.9, 544824.}, {19.91,
583862.}, {19.92, 619383.}, {19.93, 650362.}, {19.94,
672886.}, {19.95, 690179.}, {19.96, 695603.}, {19.97,
692265.}, {19.98, 677707.}, {19.99, 657226.}, {20.,
630722.}, {20.01, 599184.}, {20.02, 558854.}, {20.03,
514989.}, {20.04, 469037.}, {20.05, 421656.}, {20.06,
370503.}, {20.07, 324609.}, {20.08, 278435.}, {20.09,
233750.}, {20.1, 195167.}, {20.11, 160965.}, {20.12,
131452.}, {20.13, 108026.}, {20.14, 88341.}, {20.15,
71993.}, {20.16, 59909.}, {20.17, 51054.}, {20.18, 44365.}, {20.19,
39526.}, {20.2, 36292.}, {20.21, 34308.}, {20.22, 32666.}, {20.23,
31599.}, {20.24, 30743.}, {20.25, 29621.}, {20.26, 29034.}, {20.27,
28213.}, {20.28, 27597.}, {20.29, 27485.}, {20.3, 26921.}, {20.31,
26588.}, {20.32, 26337.}, {20.33, 25705.}, {20.34, 26199.}, {20.35,
25321.}, {20.36, 25017.}, {20.37, 25011.}, {20.38, 24566.}, {20.39,
24232.}, {20.4, 24005.}}


My starting point is



data1 = Rest@Transpose[Rescale /@ (Transpose@data)];
peakfunc[A_, [Mu]_, [Sigma]_, x_] = A^2 E^(-((x - [Mu])^2/(2 [Sigma]^2)));

Clear[model, modelvalue]
model[data_, n_] :=
Module[{dataconfig, modelfunc, objfunc, fitvar, fitres},
dataconfig = {A[#], [Mu][#], [Sigma][#]} & /@ Range[n];
modelfunc = (peakfunc[##, fitvar] & @@@ dataconfig // Total);
objfunc =
Total[((Sqrt[data[[All, 2]]])/
data[[All,
1]]) (data[[All, 2]] - (modelfunc /. fitvar -> # &) /@
data[[All, 1]])^2];
FindMinimum[objfunc, Join[{}, Flatten@dataconfig]]]
modelvalue[data_, n_] /; NumericQ[n] :=
If[n >= 1, model[data, n][[1]], 0]
fitres = ReleaseHold[
Hold[{Round[n], model[data1, Round[n]]}] /.
FindMinimum[modelvalue[data1, Round[n]], {n, 5},
Method -> "PrincipalAxis"][[2]]] // Quiet

With[{n = 2},
resfunc =
peakfunc[A[#], [Mu][#], [Sigma][#], x] & /@ Range[n] /.
model[data1, n][[2]]]

Show@{Plot[Evaluate[resfunc], {x, 0, 1},
PlotStyle -> ({Directive[Dashed, Thick,
ColorData["Rainbow"][#]]} & /@
Rescale[Range[Length[resfunc]]]), PlotRange -> All,
Frame -> True, Axes -> False],
Plot[Evaluate[Total@resfunc], {x, 0, 1},
PlotStyle -> Directive[Thick, Red], PlotRange -> All,
Frame -> True, Axes -> False],
Graphics[{PointSize[.003], Black, Point@data1}]}


This is already not very clear to me, as you can see the fit is not very good on the right side (I expect two profiles very close as the yellow and green curve in the second picture). I would like also to know how to evaluate if the fit is good enough or not.



enter image description here



enter image description here



Many thanks!







fitting






share|improve this question









New contributor




Clarine is a new contributor to this site. Take care in asking for clarification, commenting, and answering.
Check out our Code of Conduct.











share|improve this question









New contributor




Clarine is a new contributor to this site. Take care in asking for clarification, commenting, and answering.
Check out our Code of Conduct.









share|improve this question




share|improve this question








edited yesterday









Anton Antonov

22.7k164111




22.7k164111






New contributor




Clarine is a new contributor to this site. Take care in asking for clarification, commenting, and answering.
Check out our Code of Conduct.









asked yesterday









ClarineClarine

212




212




New contributor




Clarine is a new contributor to this site. Take care in asking for clarification, commenting, and answering.
Check out our Code of Conduct.





New contributor





Clarine is a new contributor to this site. Take care in asking for clarification, commenting, and answering.
Check out our Code of Conduct.






Clarine is a new contributor to this site. Take care in asking for clarification, commenting, and answering.
Check out our Code of Conduct.








  • 2




    Welcome on Mathematic.StackExchange. Maybe NonlinearModelFit might be helpful to you? At least, make sure to have a look at its documentation
    – Henrik Schumacher
    yesterday






  • 3




    As a reminder, make sure you have good initial guesses for the parameters of your fitting function.
    – J. M. is computer-less
    yesterday






  • 1




    Are the integer values counts or measurements that just recorded as integers? I ask because it is important to know if what you have is a random sample from a maybe not-so-simple probability distribution or you're performing a regression that just might happen to have the shape of a mixture of probability distributions? In other words, are there 16,928,270 observations given as a histogram? (I'm also curious as to how you might have a "feeling" about the shape of the distributions - but that's not a Mathematica issue.)
    – JimB
    yesterday












  • This question seems to be a duplicate of 98226.
    – Anton Antonov
    yesterday












  • If you do have a histogram of counts, I would have to assume that you've got a truncated sample (truncated on the left at 19.395 and truncated on the right at 20.405). If so, please add that information to the body of your question.
    – JimB
    yesterday














  • 2




    Welcome on Mathematic.StackExchange. Maybe NonlinearModelFit might be helpful to you? At least, make sure to have a look at its documentation
    – Henrik Schumacher
    yesterday






  • 3




    As a reminder, make sure you have good initial guesses for the parameters of your fitting function.
    – J. M. is computer-less
    yesterday






  • 1




    Are the integer values counts or measurements that just recorded as integers? I ask because it is important to know if what you have is a random sample from a maybe not-so-simple probability distribution or you're performing a regression that just might happen to have the shape of a mixture of probability distributions? In other words, are there 16,928,270 observations given as a histogram? (I'm also curious as to how you might have a "feeling" about the shape of the distributions - but that's not a Mathematica issue.)
    – JimB
    yesterday












  • This question seems to be a duplicate of 98226.
    – Anton Antonov
    yesterday












  • If you do have a histogram of counts, I would have to assume that you've got a truncated sample (truncated on the left at 19.395 and truncated on the right at 20.405). If so, please add that information to the body of your question.
    – JimB
    yesterday








2




2




Welcome on Mathematic.StackExchange. Maybe NonlinearModelFit might be helpful to you? At least, make sure to have a look at its documentation
– Henrik Schumacher
yesterday




Welcome on Mathematic.StackExchange. Maybe NonlinearModelFit might be helpful to you? At least, make sure to have a look at its documentation
– Henrik Schumacher
yesterday




3




3




As a reminder, make sure you have good initial guesses for the parameters of your fitting function.
– J. M. is computer-less
yesterday




As a reminder, make sure you have good initial guesses for the parameters of your fitting function.
– J. M. is computer-less
yesterday




1




1




Are the integer values counts or measurements that just recorded as integers? I ask because it is important to know if what you have is a random sample from a maybe not-so-simple probability distribution or you're performing a regression that just might happen to have the shape of a mixture of probability distributions? In other words, are there 16,928,270 observations given as a histogram? (I'm also curious as to how you might have a "feeling" about the shape of the distributions - but that's not a Mathematica issue.)
– JimB
yesterday






Are the integer values counts or measurements that just recorded as integers? I ask because it is important to know if what you have is a random sample from a maybe not-so-simple probability distribution or you're performing a regression that just might happen to have the shape of a mixture of probability distributions? In other words, are there 16,928,270 observations given as a histogram? (I'm also curious as to how you might have a "feeling" about the shape of the distributions - but that's not a Mathematica issue.)
– JimB
yesterday














This question seems to be a duplicate of 98226.
– Anton Antonov
yesterday






This question seems to be a duplicate of 98226.
– Anton Antonov
yesterday














If you do have a histogram of counts, I would have to assume that you've got a truncated sample (truncated on the left at 19.395 and truncated on the right at 20.405). If so, please add that information to the body of your question.
– JimB
yesterday




If you do have a histogram of counts, I would have to assume that you've got a truncated sample (truncated on the left at 19.395 and truncated on the right at 20.405). If so, please add that information to the body of your question.
– JimB
yesterday










1 Answer
1






active

oldest

votes


















6














Here is a fit using Fit and a basis of functions. I am not sure is this a solution OP wants -- may be only one Exp term is desired.



(The data variable data is taken from the question.)



(* Taken from the question. *)
Clear[peakfunc]
peakfunc[A_, [Mu]_, [Sigma]_, x_] = A^2 E^(-((x - [Mu])^2/(2 [Sigma]^2)));

(* Generate a basis of functions. *)
bFuncs = Flatten@
Table[peakfunc[1, m, s, x], {m, 19.4, 20.4, 0.2}, {s, 0.1, 0.4, 0.1}];
Length[bFuncs]
(* 24 *)

(* Load the QRMon package. *)
Import["https://raw.githubusercontent.com/antononcube/MathematicaForPrediction/master/MonadicProgramming/MonadicQuantileRegression.m"]

(* Do a fit with the basis, plot it, and plot the relative errors. *)
qmon =
QRMonUnit[data]⟹
QRMonEchoDataSummary⟹
QRMonFit[bFuncs]⟹
QRMonPlot⟹
QRMonErrorPlots;


enter image description here



(* Take the regression function from the monad object and show it. *)
qFunc = (qmon⟹QRMonTakeRegressionFunctions)["mean"];
qFunc[x]

(* -2.445*10^7 E^(-50. (-20.4 + x)^2) +
3.41622*10^10 E^(-12.5 (-20.4 + x)^2) -
1.63757*10^11 E^(-5.55556 (-20.4 + x)^2) -
2.91489*10^11 E^(-3.125 (-20.4 + x)^2) +
1.83408*10^7 E^(-50. (-20.2 + x)^2) +
6.05021*10^10 E^(-12.5 (-20.2 + x)^2) -
4.04765*10^11 E^(-5.55556 (-20.2 + x)^2) +
8.90334*10^11 E^(-3.125 (-20.2 + x)^2) -
1.47152*10^7 E^(-50. (-20. + x)^2) +
5.14204*10^10 E^(-12.5 (-20. + x)^2) -
1.34254*10^11 E^(-5.55556 (-20. + x)^2) +
1.54544*10^11 E^(-3.125 (-20. + x)^2) +
1.35117*10^7 E^(-50. (-19.8 + x)^2) +
1.77025*10^10 E^(-12.5 (-19.8 + x)^2) -
2.38068*10^10 E^(-5.55556 (-19.8 + x)^2) -
1.63691*10^11 E^(-3.125 (-19.8 + x)^2) -
1.75233*10^7 E^(-50. (-19.6 + x)^2) -
1.74451*10^10 E^(-12.5 (-19.6 + x)^2) +
2.91813*10^11 E^(-5.55556 (-19.6 + x)^2) -
6.2293*10^11 E^(-3.125 (-19.6 + x)^2) +
3.084*10^7 E^(-50. (-19.4 + x)^2) -
2.02534*10^10 E^(-12.5 (-19.4 + x)^2) +
1.21184*10^11 E^(-5.55556 (-19.4 + x)^2) +
2.04278*10^11 E^(-3.125 (-19.4 + x)^2) *)





share|improve this answer























  • First, thank you for the answer. Is it for a multi peak fitting? Where you define the number of peak that you think there are in your set of data?
    – Clarine
    5 hours ago










  • @Clarine Your comment indicates that you have a different question in mind that the one you posted. Is this discussion (175989) more in line if with what you are looking for?
    – Anton Antonov
    4 hours ago












  • Yes and no, so the thing is that I believe this dataset could be fit by two peaks (voigt, lorentzian, Gaussian or maybe a combination) but the problem is that I don't know how to write where I believe the centre of the peak is. What I want to do is basically sat "ok we have this peak, I want to try and fit these like we have two peaks and not only one" and check whether my guess makes sense. In the case that you have linked the peak positions were clear
    – Clarine
    16 mins ago











Your Answer





StackExchange.ifUsing("editor", function () {
return StackExchange.using("mathjaxEditing", function () {
StackExchange.MarkdownEditor.creationCallbacks.add(function (editor, postfix) {
StackExchange.mathjaxEditing.prepareWmdForMathJax(editor, postfix, [["$", "$"], ["\\(","\\)"]]);
});
});
}, "mathjax-editing");

StackExchange.ready(function() {
var channelOptions = {
tags: "".split(" "),
id: "387"
};
initTagRenderer("".split(" "), "".split(" "), channelOptions);

StackExchange.using("externalEditor", function() {
// Have to fire editor after snippets, if snippets enabled
if (StackExchange.settings.snippets.snippetsEnabled) {
StackExchange.using("snippets", function() {
createEditor();
});
}
else {
createEditor();
}
});

function createEditor() {
StackExchange.prepareEditor({
heartbeatType: 'answer',
autoActivateHeartbeat: false,
convertImagesToLinks: false,
noModals: true,
showLowRepImageUploadWarning: true,
reputationToPostImages: null,
bindNavPrevention: true,
postfix: "",
imageUploader: {
brandingHtml: "Powered by u003ca class="icon-imgur-white" href="https://imgur.com/"u003eu003c/au003e",
contentPolicyHtml: "User contributions licensed under u003ca href="https://creativecommons.org/licenses/by-sa/3.0/"u003ecc by-sa 3.0 with attribution requiredu003c/au003e u003ca href="https://stackoverflow.com/legal/content-policy"u003e(content policy)u003c/au003e",
allowUrls: true
},
onDemand: true,
discardSelector: ".discard-answer"
,immediatelyShowMarkdownHelp:true
});


}
});






Clarine is a new contributor. Be nice, and check out our Code of Conduct.










draft saved

draft discarded


















StackExchange.ready(
function () {
StackExchange.openid.initPostLogin('.new-post-login', 'https%3a%2f%2fmathematica.stackexchange.com%2fquestions%2f188934%2fmulti-peaks-fit-voigt-lorentzian-or-gaussian%23new-answer', 'question_page');
}
);

Post as a guest















Required, but never shown

























1 Answer
1






active

oldest

votes








1 Answer
1






active

oldest

votes









active

oldest

votes






active

oldest

votes









6














Here is a fit using Fit and a basis of functions. I am not sure is this a solution OP wants -- may be only one Exp term is desired.



(The data variable data is taken from the question.)



(* Taken from the question. *)
Clear[peakfunc]
peakfunc[A_, [Mu]_, [Sigma]_, x_] = A^2 E^(-((x - [Mu])^2/(2 [Sigma]^2)));

(* Generate a basis of functions. *)
bFuncs = Flatten@
Table[peakfunc[1, m, s, x], {m, 19.4, 20.4, 0.2}, {s, 0.1, 0.4, 0.1}];
Length[bFuncs]
(* 24 *)

(* Load the QRMon package. *)
Import["https://raw.githubusercontent.com/antononcube/MathematicaForPrediction/master/MonadicProgramming/MonadicQuantileRegression.m"]

(* Do a fit with the basis, plot it, and plot the relative errors. *)
qmon =
QRMonUnit[data]⟹
QRMonEchoDataSummary⟹
QRMonFit[bFuncs]⟹
QRMonPlot⟹
QRMonErrorPlots;


enter image description here



(* Take the regression function from the monad object and show it. *)
qFunc = (qmon⟹QRMonTakeRegressionFunctions)["mean"];
qFunc[x]

(* -2.445*10^7 E^(-50. (-20.4 + x)^2) +
3.41622*10^10 E^(-12.5 (-20.4 + x)^2) -
1.63757*10^11 E^(-5.55556 (-20.4 + x)^2) -
2.91489*10^11 E^(-3.125 (-20.4 + x)^2) +
1.83408*10^7 E^(-50. (-20.2 + x)^2) +
6.05021*10^10 E^(-12.5 (-20.2 + x)^2) -
4.04765*10^11 E^(-5.55556 (-20.2 + x)^2) +
8.90334*10^11 E^(-3.125 (-20.2 + x)^2) -
1.47152*10^7 E^(-50. (-20. + x)^2) +
5.14204*10^10 E^(-12.5 (-20. + x)^2) -
1.34254*10^11 E^(-5.55556 (-20. + x)^2) +
1.54544*10^11 E^(-3.125 (-20. + x)^2) +
1.35117*10^7 E^(-50. (-19.8 + x)^2) +
1.77025*10^10 E^(-12.5 (-19.8 + x)^2) -
2.38068*10^10 E^(-5.55556 (-19.8 + x)^2) -
1.63691*10^11 E^(-3.125 (-19.8 + x)^2) -
1.75233*10^7 E^(-50. (-19.6 + x)^2) -
1.74451*10^10 E^(-12.5 (-19.6 + x)^2) +
2.91813*10^11 E^(-5.55556 (-19.6 + x)^2) -
6.2293*10^11 E^(-3.125 (-19.6 + x)^2) +
3.084*10^7 E^(-50. (-19.4 + x)^2) -
2.02534*10^10 E^(-12.5 (-19.4 + x)^2) +
1.21184*10^11 E^(-5.55556 (-19.4 + x)^2) +
2.04278*10^11 E^(-3.125 (-19.4 + x)^2) *)





share|improve this answer























  • First, thank you for the answer. Is it for a multi peak fitting? Where you define the number of peak that you think there are in your set of data?
    – Clarine
    5 hours ago










  • @Clarine Your comment indicates that you have a different question in mind that the one you posted. Is this discussion (175989) more in line if with what you are looking for?
    – Anton Antonov
    4 hours ago












  • Yes and no, so the thing is that I believe this dataset could be fit by two peaks (voigt, lorentzian, Gaussian or maybe a combination) but the problem is that I don't know how to write where I believe the centre of the peak is. What I want to do is basically sat "ok we have this peak, I want to try and fit these like we have two peaks and not only one" and check whether my guess makes sense. In the case that you have linked the peak positions were clear
    – Clarine
    16 mins ago
















6














Here is a fit using Fit and a basis of functions. I am not sure is this a solution OP wants -- may be only one Exp term is desired.



(The data variable data is taken from the question.)



(* Taken from the question. *)
Clear[peakfunc]
peakfunc[A_, [Mu]_, [Sigma]_, x_] = A^2 E^(-((x - [Mu])^2/(2 [Sigma]^2)));

(* Generate a basis of functions. *)
bFuncs = Flatten@
Table[peakfunc[1, m, s, x], {m, 19.4, 20.4, 0.2}, {s, 0.1, 0.4, 0.1}];
Length[bFuncs]
(* 24 *)

(* Load the QRMon package. *)
Import["https://raw.githubusercontent.com/antononcube/MathematicaForPrediction/master/MonadicProgramming/MonadicQuantileRegression.m"]

(* Do a fit with the basis, plot it, and plot the relative errors. *)
qmon =
QRMonUnit[data]⟹
QRMonEchoDataSummary⟹
QRMonFit[bFuncs]⟹
QRMonPlot⟹
QRMonErrorPlots;


enter image description here



(* Take the regression function from the monad object and show it. *)
qFunc = (qmon⟹QRMonTakeRegressionFunctions)["mean"];
qFunc[x]

(* -2.445*10^7 E^(-50. (-20.4 + x)^2) +
3.41622*10^10 E^(-12.5 (-20.4 + x)^2) -
1.63757*10^11 E^(-5.55556 (-20.4 + x)^2) -
2.91489*10^11 E^(-3.125 (-20.4 + x)^2) +
1.83408*10^7 E^(-50. (-20.2 + x)^2) +
6.05021*10^10 E^(-12.5 (-20.2 + x)^2) -
4.04765*10^11 E^(-5.55556 (-20.2 + x)^2) +
8.90334*10^11 E^(-3.125 (-20.2 + x)^2) -
1.47152*10^7 E^(-50. (-20. + x)^2) +
5.14204*10^10 E^(-12.5 (-20. + x)^2) -
1.34254*10^11 E^(-5.55556 (-20. + x)^2) +
1.54544*10^11 E^(-3.125 (-20. + x)^2) +
1.35117*10^7 E^(-50. (-19.8 + x)^2) +
1.77025*10^10 E^(-12.5 (-19.8 + x)^2) -
2.38068*10^10 E^(-5.55556 (-19.8 + x)^2) -
1.63691*10^11 E^(-3.125 (-19.8 + x)^2) -
1.75233*10^7 E^(-50. (-19.6 + x)^2) -
1.74451*10^10 E^(-12.5 (-19.6 + x)^2) +
2.91813*10^11 E^(-5.55556 (-19.6 + x)^2) -
6.2293*10^11 E^(-3.125 (-19.6 + x)^2) +
3.084*10^7 E^(-50. (-19.4 + x)^2) -
2.02534*10^10 E^(-12.5 (-19.4 + x)^2) +
1.21184*10^11 E^(-5.55556 (-19.4 + x)^2) +
2.04278*10^11 E^(-3.125 (-19.4 + x)^2) *)





share|improve this answer























  • First, thank you for the answer. Is it for a multi peak fitting? Where you define the number of peak that you think there are in your set of data?
    – Clarine
    5 hours ago










  • @Clarine Your comment indicates that you have a different question in mind that the one you posted. Is this discussion (175989) more in line if with what you are looking for?
    – Anton Antonov
    4 hours ago












  • Yes and no, so the thing is that I believe this dataset could be fit by two peaks (voigt, lorentzian, Gaussian or maybe a combination) but the problem is that I don't know how to write where I believe the centre of the peak is. What I want to do is basically sat "ok we have this peak, I want to try and fit these like we have two peaks and not only one" and check whether my guess makes sense. In the case that you have linked the peak positions were clear
    – Clarine
    16 mins ago














6












6








6






Here is a fit using Fit and a basis of functions. I am not sure is this a solution OP wants -- may be only one Exp term is desired.



(The data variable data is taken from the question.)



(* Taken from the question. *)
Clear[peakfunc]
peakfunc[A_, [Mu]_, [Sigma]_, x_] = A^2 E^(-((x - [Mu])^2/(2 [Sigma]^2)));

(* Generate a basis of functions. *)
bFuncs = Flatten@
Table[peakfunc[1, m, s, x], {m, 19.4, 20.4, 0.2}, {s, 0.1, 0.4, 0.1}];
Length[bFuncs]
(* 24 *)

(* Load the QRMon package. *)
Import["https://raw.githubusercontent.com/antononcube/MathematicaForPrediction/master/MonadicProgramming/MonadicQuantileRegression.m"]

(* Do a fit with the basis, plot it, and plot the relative errors. *)
qmon =
QRMonUnit[data]⟹
QRMonEchoDataSummary⟹
QRMonFit[bFuncs]⟹
QRMonPlot⟹
QRMonErrorPlots;


enter image description here



(* Take the regression function from the monad object and show it. *)
qFunc = (qmon⟹QRMonTakeRegressionFunctions)["mean"];
qFunc[x]

(* -2.445*10^7 E^(-50. (-20.4 + x)^2) +
3.41622*10^10 E^(-12.5 (-20.4 + x)^2) -
1.63757*10^11 E^(-5.55556 (-20.4 + x)^2) -
2.91489*10^11 E^(-3.125 (-20.4 + x)^2) +
1.83408*10^7 E^(-50. (-20.2 + x)^2) +
6.05021*10^10 E^(-12.5 (-20.2 + x)^2) -
4.04765*10^11 E^(-5.55556 (-20.2 + x)^2) +
8.90334*10^11 E^(-3.125 (-20.2 + x)^2) -
1.47152*10^7 E^(-50. (-20. + x)^2) +
5.14204*10^10 E^(-12.5 (-20. + x)^2) -
1.34254*10^11 E^(-5.55556 (-20. + x)^2) +
1.54544*10^11 E^(-3.125 (-20. + x)^2) +
1.35117*10^7 E^(-50. (-19.8 + x)^2) +
1.77025*10^10 E^(-12.5 (-19.8 + x)^2) -
2.38068*10^10 E^(-5.55556 (-19.8 + x)^2) -
1.63691*10^11 E^(-3.125 (-19.8 + x)^2) -
1.75233*10^7 E^(-50. (-19.6 + x)^2) -
1.74451*10^10 E^(-12.5 (-19.6 + x)^2) +
2.91813*10^11 E^(-5.55556 (-19.6 + x)^2) -
6.2293*10^11 E^(-3.125 (-19.6 + x)^2) +
3.084*10^7 E^(-50. (-19.4 + x)^2) -
2.02534*10^10 E^(-12.5 (-19.4 + x)^2) +
1.21184*10^11 E^(-5.55556 (-19.4 + x)^2) +
2.04278*10^11 E^(-3.125 (-19.4 + x)^2) *)





share|improve this answer














Here is a fit using Fit and a basis of functions. I am not sure is this a solution OP wants -- may be only one Exp term is desired.



(The data variable data is taken from the question.)



(* Taken from the question. *)
Clear[peakfunc]
peakfunc[A_, [Mu]_, [Sigma]_, x_] = A^2 E^(-((x - [Mu])^2/(2 [Sigma]^2)));

(* Generate a basis of functions. *)
bFuncs = Flatten@
Table[peakfunc[1, m, s, x], {m, 19.4, 20.4, 0.2}, {s, 0.1, 0.4, 0.1}];
Length[bFuncs]
(* 24 *)

(* Load the QRMon package. *)
Import["https://raw.githubusercontent.com/antononcube/MathematicaForPrediction/master/MonadicProgramming/MonadicQuantileRegression.m"]

(* Do a fit with the basis, plot it, and plot the relative errors. *)
qmon =
QRMonUnit[data]⟹
QRMonEchoDataSummary⟹
QRMonFit[bFuncs]⟹
QRMonPlot⟹
QRMonErrorPlots;


enter image description here



(* Take the regression function from the monad object and show it. *)
qFunc = (qmon⟹QRMonTakeRegressionFunctions)["mean"];
qFunc[x]

(* -2.445*10^7 E^(-50. (-20.4 + x)^2) +
3.41622*10^10 E^(-12.5 (-20.4 + x)^2) -
1.63757*10^11 E^(-5.55556 (-20.4 + x)^2) -
2.91489*10^11 E^(-3.125 (-20.4 + x)^2) +
1.83408*10^7 E^(-50. (-20.2 + x)^2) +
6.05021*10^10 E^(-12.5 (-20.2 + x)^2) -
4.04765*10^11 E^(-5.55556 (-20.2 + x)^2) +
8.90334*10^11 E^(-3.125 (-20.2 + x)^2) -
1.47152*10^7 E^(-50. (-20. + x)^2) +
5.14204*10^10 E^(-12.5 (-20. + x)^2) -
1.34254*10^11 E^(-5.55556 (-20. + x)^2) +
1.54544*10^11 E^(-3.125 (-20. + x)^2) +
1.35117*10^7 E^(-50. (-19.8 + x)^2) +
1.77025*10^10 E^(-12.5 (-19.8 + x)^2) -
2.38068*10^10 E^(-5.55556 (-19.8 + x)^2) -
1.63691*10^11 E^(-3.125 (-19.8 + x)^2) -
1.75233*10^7 E^(-50. (-19.6 + x)^2) -
1.74451*10^10 E^(-12.5 (-19.6 + x)^2) +
2.91813*10^11 E^(-5.55556 (-19.6 + x)^2) -
6.2293*10^11 E^(-3.125 (-19.6 + x)^2) +
3.084*10^7 E^(-50. (-19.4 + x)^2) -
2.02534*10^10 E^(-12.5 (-19.4 + x)^2) +
1.21184*10^11 E^(-5.55556 (-19.4 + x)^2) +
2.04278*10^11 E^(-3.125 (-19.4 + x)^2) *)






share|improve this answer














share|improve this answer



share|improve this answer








edited yesterday

























answered yesterday









Anton AntonovAnton Antonov

22.7k164111




22.7k164111












  • First, thank you for the answer. Is it for a multi peak fitting? Where you define the number of peak that you think there are in your set of data?
    – Clarine
    5 hours ago










  • @Clarine Your comment indicates that you have a different question in mind that the one you posted. Is this discussion (175989) more in line if with what you are looking for?
    – Anton Antonov
    4 hours ago












  • Yes and no, so the thing is that I believe this dataset could be fit by two peaks (voigt, lorentzian, Gaussian or maybe a combination) but the problem is that I don't know how to write where I believe the centre of the peak is. What I want to do is basically sat "ok we have this peak, I want to try and fit these like we have two peaks and not only one" and check whether my guess makes sense. In the case that you have linked the peak positions were clear
    – Clarine
    16 mins ago


















  • First, thank you for the answer. Is it for a multi peak fitting? Where you define the number of peak that you think there are in your set of data?
    – Clarine
    5 hours ago










  • @Clarine Your comment indicates that you have a different question in mind that the one you posted. Is this discussion (175989) more in line if with what you are looking for?
    – Anton Antonov
    4 hours ago












  • Yes and no, so the thing is that I believe this dataset could be fit by two peaks (voigt, lorentzian, Gaussian or maybe a combination) but the problem is that I don't know how to write where I believe the centre of the peak is. What I want to do is basically sat "ok we have this peak, I want to try and fit these like we have two peaks and not only one" and check whether my guess makes sense. In the case that you have linked the peak positions were clear
    – Clarine
    16 mins ago
















First, thank you for the answer. Is it for a multi peak fitting? Where you define the number of peak that you think there are in your set of data?
– Clarine
5 hours ago




First, thank you for the answer. Is it for a multi peak fitting? Where you define the number of peak that you think there are in your set of data?
– Clarine
5 hours ago












@Clarine Your comment indicates that you have a different question in mind that the one you posted. Is this discussion (175989) more in line if with what you are looking for?
– Anton Antonov
4 hours ago






@Clarine Your comment indicates that you have a different question in mind that the one you posted. Is this discussion (175989) more in line if with what you are looking for?
– Anton Antonov
4 hours ago














Yes and no, so the thing is that I believe this dataset could be fit by two peaks (voigt, lorentzian, Gaussian or maybe a combination) but the problem is that I don't know how to write where I believe the centre of the peak is. What I want to do is basically sat "ok we have this peak, I want to try and fit these like we have two peaks and not only one" and check whether my guess makes sense. In the case that you have linked the peak positions were clear
– Clarine
16 mins ago




Yes and no, so the thing is that I believe this dataset could be fit by two peaks (voigt, lorentzian, Gaussian or maybe a combination) but the problem is that I don't know how to write where I believe the centre of the peak is. What I want to do is basically sat "ok we have this peak, I want to try and fit these like we have two peaks and not only one" and check whether my guess makes sense. In the case that you have linked the peak positions were clear
– Clarine
16 mins ago










Clarine is a new contributor. Be nice, and check out our Code of Conduct.










draft saved

draft discarded


















Clarine is a new contributor. Be nice, and check out our Code of Conduct.













Clarine is a new contributor. Be nice, and check out our Code of Conduct.












Clarine is a new contributor. Be nice, and check out our Code of Conduct.
















Thanks for contributing an answer to Mathematica Stack Exchange!


  • Please be sure to answer the question. Provide details and share your research!

But avoid



  • Asking for help, clarification, or responding to other answers.

  • Making statements based on opinion; back them up with references or personal experience.


Use MathJax to format equations. MathJax reference.


To learn more, see our tips on writing great answers.





Some of your past answers have not been well-received, and you're in danger of being blocked from answering.


Please pay close attention to the following guidance:


  • Please be sure to answer the question. Provide details and share your research!

But avoid



  • Asking for help, clarification, or responding to other answers.

  • Making statements based on opinion; back them up with references or personal experience.


To learn more, see our tips on writing great answers.




draft saved


draft discarded














StackExchange.ready(
function () {
StackExchange.openid.initPostLogin('.new-post-login', 'https%3a%2f%2fmathematica.stackexchange.com%2fquestions%2f188934%2fmulti-peaks-fit-voigt-lorentzian-or-gaussian%23new-answer', 'question_page');
}
);

Post as a guest















Required, but never shown





















































Required, but never shown














Required, but never shown












Required, but never shown







Required, but never shown

































Required, but never shown














Required, but never shown












Required, but never shown







Required, but never shown







Popular posts from this blog

An IMO inspired problem

Management

Has there ever been an instance of an active nuclear power plant within or near a war zone?